Difference between revisions of "2019 AMC 8 Problems/Problem 23"

(Solution 3)
m (Video explaining solution:)
Line 13: Line 13:
 
Fakesolve: We first start by setting the total number of points as <math>28</math>, since <math>\text{lcm}(4,7) = 28</math>. However, we see that this does not work since we surpass the number of points just with the information given (<math>28\cdot\frac{1}{4}+28\cdot\frac{2}{7} + 15 > 28</math>). Next, we assume that the total number of points scored is <math>56</math>. With this, we have that Alexa, Brittany, and Chelsea score: <math>56\cdot\frac{1}{4}+56\cdot\frac{2}{7} + 15 = 45</math>, and thus, the other seven players would have scored a total of <math>56-45 = \boxed{\textbf{(B)} 11}</math> (We see that this works since we could have <math>4</math> of them score <math>2</math> points, and the other <math>3</math> of them score <math>1</math> point) -aops5234
 
Fakesolve: We first start by setting the total number of points as <math>28</math>, since <math>\text{lcm}(4,7) = 28</math>. However, we see that this does not work since we surpass the number of points just with the information given (<math>28\cdot\frac{1}{4}+28\cdot\frac{2}{7} + 15 > 28</math>). Next, we assume that the total number of points scored is <math>56</math>. With this, we have that Alexa, Brittany, and Chelsea score: <math>56\cdot\frac{1}{4}+56\cdot\frac{2}{7} + 15 = 45</math>, and thus, the other seven players would have scored a total of <math>56-45 = \boxed{\textbf{(B)} 11}</math> (We see that this works since we could have <math>4</math> of them score <math>2</math> points, and the other <math>3</math> of them score <math>1</math> point) -aops5234
  
==Video explaining solution:==  
+
==Video explaining solution==  
  
 
https://youtu.be/wsYCn2FqZJE
 
https://youtu.be/wsYCn2FqZJE

Revision as of 15:11, 6 May 2020

Problem 23

After Euclid High School's last basketball game, it was determined that $\frac{1}{4}$ of the team's points were scored by Alexa and $\frac{2}{7}$ were scored by Brittany. Chelsea scored $15$ points. None of the other $7$ team members scored more than $2$ points What was the total number of points scored by the other $7$ team members?

$\textbf{(A) }10\qquad\textbf{(B) }11\qquad\textbf{(C) }12\qquad\textbf{(D) }13\qquad\textbf{(E) }14$

Solution 1

Since $\frac{\text{total points}}{4}$ and $\frac{2(\text{total points})}{7}$ are integers, we have $28 | \text{total points}$. We see that the number of points scored by the other team members is less than or equal to $14$ and greater than or equal to $0$. We let the total number of points be $t$ and the total number of points scored by the other team members be $x$, which means that $\frac{t}{4} + \frac{2t}{7} + 15 + x = t \quad \implies \quad 0 \le \frac{13t}{28} - 15 = x \le 14$, which means $15 \le \frac{13t}{28} \le 29$. The only value of $t$ that satisfies all conditions listed is $56$, so $x=\boxed{\textbf{(B)} 11}$. - juliankuang (lol im smart.)

Solution 2

Starting from the above equation $\frac{t}{4}+\frac{2t}{7} + 15 + x = t$ where $t$ is the total number of points scored and $x\le 14$ is the number of points scored by the remaining 7 team members, we can simplify to obtain the Diophantine equation $x+15 = \frac{13}{28}t$, or $28x+28\cdot 15=13t$. Since $t$ is necessarily divisible by 28, let $t=28u$ where $u \ge 0$ and divide by 28 to obtain $x + 15 = 13u$. Then it is easy to see $u=2$ ($t=56$) is the only candidate, giving $x=\boxed{\textbf{(B)} 11}$. -scrabbler94

Solution 3

Fakesolve: We first start by setting the total number of points as $28$, since $\text{lcm}(4,7) = 28$. However, we see that this does not work since we surpass the number of points just with the information given ($28\cdot\frac{1}{4}+28\cdot\frac{2}{7} + 15 > 28$). Next, we assume that the total number of points scored is $56$. With this, we have that Alexa, Brittany, and Chelsea score: $56\cdot\frac{1}{4}+56\cdot\frac{2}{7} + 15 = 45$, and thus, the other seven players would have scored a total of $56-45 = \boxed{\textbf{(B)} 11}$ (We see that this works since we could have $4$ of them score $2$ points, and the other $3$ of them score $1$ point) -aops5234

Video explaining solution

https://youtu.be/wsYCn2FqZJE

https://www.youtube.com/watch?v=fKjmw_zzCUU

See Also

2019 AMC 8 (ProblemsAnswer KeyResources)
Preceded by
Problem 22
Followed by
Problem 24
1 2 3 4 5 6 7 8 9 10 11 12 13 14 15 16 17 18 19 20 21 22 23 24 25
All AJHSME/AMC 8 Problems and Solutions

The problems on this page are copyrighted by the Mathematical Association of America's American Mathematics Competitions. AMC logo.png